Generating set for finitely generated submodule of finitely generated module

Let $R$ be a ring,  $M$ a finitely generated $R$-module, and $N \subseteq M$ a f.g. submodule. Suppose $M$ has a minimal generating set of $m$ elements, then under what conditions may we assume that $N$ has a (not necessarily minimal) generating set of $n \leq m$ generators? I know the result holds, for example, when $R$ is a PID, but are there any other sufficient (or even necessary) conditions on $R$ (and/or $M,N$) to provide a complete characterization? A reference would be welcome.

Answer

Answers can only be viewed under the following conditions:
  1. The questioner was satisfied with and accepted the answer, or
  2. The answer was evaluated as being 100% correct by the judge.
View the answer

1 Attachment

Erdos Erdos
4.7K
The answer is accepted.
Join Matchmaticians Affiliate Marketing Program to earn up to a 50% commission on every question that your affiliated users ask or answer.